« first day (4691 days earlier)      last day (332 days later) » 

7:00 AM
@shintuku good night.
 
7:13 AM
@Koro $A, B\subset X$ are separated iff $cl(A) \cap B=\emptyset=A\cap cl(B) $
 
yes, but that's not what is meant here.
it is meant in the sense that: path connected components of topologist's sine curve are not separated from each other.
by tsc here I mean: union of sin 1/x graph with {0}\times [-1,1].
apparently, this means that union of two pcc need not be disconnected.
 
@Koro It pretty much depends on context.I have given the usual definition of "separated sets" or "strongly disjoint sets" in a topological space.
@Koro This is true for any connected space.
 
yeah, I know this definition. But it doesn't seem to fit here.
 
As $X$ is union of it's path components.
 
@SouravGhosh oh right.
noting that path components lie in connected somponents.
but still it doesn't feel right.
 
7:29 AM
$x~y$ iff there is a path connecting $x$ and $y$.
 
@SouravGhosh shouldn't we say that 'for any connected space with exactly two path components'?
but if there are say infinitely many path components, then union of any two may be disconnected.
I mean what prevents that really?
 
@Shaun:
 
in case of only two, this does not happen.
 
0
Q: Bizarre function that generalizes the inclusion-exclusion formula for $\pi(t) - \pi(\sqrt{t + 1})$. For all reals $t\geq 5$, the function is non-zero

Daniel Donnelly Conjecture. The following arithmetic function is never zero, for any $t \in \Bbb{R}$, and $t \geq 5$: $$ g(t) := \sum_{d \mid p_n\#}(-1)^{\omega(d)}\lfloor\frac{t}{d}\rfloor|G_d| $$ where $G_d = \{ x \in \Bbb{Z}/d : x^2 = 1\pmod d\}$, and where $n(x) := \pi(\sqrt{x + 1})$ varies with $x$. What...

 
Hi DLeftadjoint
 
7:38 AM
@Koro HI, I invented that formula
it's got groups in it
It's weird, right?
Thx, bra
 
@Koro Yes. Choose three points discrete space.
 
@SouravGhosh no, we want a connected space example.
@SouravGhosh i.e. a counterexample to this.
counterexample if any will show up only if we have more than 2 path components.
 
@Koro, want to learn more about my formula?
It has groups!
Subgroups of the group of units modulo square-free $d$
Square-free just means not divisible by any $q^2$ prime squared
 
not right now
:(
 
:'(
All ops needed to know formula: floor, +, -, /, *, |.| (group order) and # (primorial)
 
7:49 AM
If $A$ is measurable subset of R of measure $>0$, then there is a $d>0$ such that $A\cap (A+x)$ is non empty whenever $|x|<d$.
how to prove this?
If I show that f(x)= m(A\cap (A+x)) is continuous at 0 then I am done.
But I'm having difficulty in showing that: $f(x)- f(0)= m(A\cap (A+x))- m(A)\le |A|-|A|=0$
how to show $f(0)-f(x)$ is also small when x is near 0?
 
@Koro Discrete space is an example where union of two path component is always disconnected.
So we need an example of a topological space where union of two path components is always connected.
 
no, we need an example of a connected space in which union of two path components is NOT connected.
@SouravGhosh this is topologist's sine curve.
12 mins ago, by Koro
counterexample if any will show up only if we have more than 2 path components.
 
@Koro Trivial for connected space with exactly two path compoents.
 
indeed
 
@Koro Trivial for connected space with exactly two path components.
 
7:58 AM
Assume $0<a_n \le a_{n+1}$ for each $n\in\mathbb{N}$, show that $\sum_{n=1}^\infty a_n$ diverges. My attempt: since $(a_n)_n$ is increasing, its limit exists with $\lim_{n \to +\infty} a_n= \sup_{n\in\mathbb{N}} a_n$. But $\sup_{n\in\mathbb{N}} a_n \ge a_n >0$, hence $\lim_{n \to +\infty} a_n>0$. This implies that the necessary condition for convergence is not satisfied. Does this work?
 
@ZaWarudo you are trying to prove a wrong statement.
@SouravGhosh more trivial for a connected space with exactly one path component. 😅
oh, my bad @ZaWarudo. I didn't see $\sum_{n=1}^\infty$ there.
I misunderstood that as $\lim a_n$.
 
@Koro: No worries, thank you for the reply :) do you think my proof works?
 
Let me take a look at it. (Earlier, I didn't see it after what I thought to be $\lim$ which was actually a sum.)
@ZaWarudo it looks fine to me.
although for definiteness, you could have said: $\lim a_n=\sup a_n>= a_1>0$ or add ' for all n' after your 'But $\sup a_n \ge a_n\gt 0$'.
 
Yes, indeed I was just thinking that maybe it would be better to fix the estimation from below with the first term. Thanks again :)
Is this necessary because from $a_n>0$ I can only deduce $\lim_{n \to +\infty} a_n \ge 0$, right?
While saying $a_n \ge a_1>0$ implies $\lim_{n \to +\infty} a_n \ge a_1>0$ and so $\lim_{n \to +\infty} a_n>0$
 
8:14 AM
if you want to 'optimize' this proof by making it shorter or involving less things, it's maybe worth thinking about your 'necessary condition for convergence.' to deduce divergence, it likely doesn't require that lim a_n exist. it likely only requires you to know that it is not the case that lim a_n = 0. you could write a proof with fewer inferences from this, if you want to take that test as a given.
all of this stuff is only a few lines removed from the definitions, so you might also try a proof from the definitions (i.e., not routing through the test for divergence)
 
@leslietownes You mean something like this: $\sum_{n=1}^\infty =\lim_{N\to+\infty} \sum_{n=1}^N a_n \ge \lim_{N\to+\infty} \sum_{n=1}^N a_1=\lim_{n\to+\infty} Na_1=+\infty$? Here, I only used that $0<a_n \le a_{n+1}$ for each $n\in\mathbb{N}$ (to say that $a_1>0$ and so the latter limit is $+\infty$) and I am not using the existence of $\lim_{n \to +\infty} a_n$.
Sorry, I edited because I forgot a sum :D
 
i was thinking of a proof that began "let e > 0 be given." just for fun. but yes, any number of arguments like that will work too.
its helpful to be able use basic results (like the 'test for divergence') as black boxes in arguments, because that tracks how we often make use of results that we 'know' after we know them. it's also helpful to be able to write proofs of this kind of stuff from the definitions, because when it's possible to do so, as it is here. the arguments used to establish all of those basic results about the algebra of limits are just as useful as the results themselves.
but yeah, your formulation above is interesting because it proves a more general theorem (i.e., that if a_n is a sequence satisfying a_n >= a_1 > 0 for all n, then sum a_n diverges) and without making use of the existence of lim a_n in the case where a_n happens to be increasing.
you're still kinda using the extended real numbers to assign a "value" +infty to lim N a_1 there, and some theorems about what happens to inequalities when you pass to a limit there, and if you wanted to you could remove those things too.
its just fun to look at these kinds of things from multiple perspectives.
 
Mad
8:34 AM
hello, i am having issues with notation of cyclic sums.
Suppose i have two numbers a_i b_i that are cyclic in i =0...n such that n+m = m for m = 0...n
I want to show that the sum from j = 1 to n of a_j-1 b_j is the same as the sum of j=1 to n of a_j b_j-1
more over , consider them not to be numbers, but a as a vector and b as a linear map
 
@TedShifrin I usually do that, but I want to know why using the rank's notion doesn't lead to satisfactory results. May be I'm wrong after all.
 
8:57 AM
mad: by 'a is cyclic' do you mean that a_{n+m} = a_m for all m and similarly for b? (you've written just "n+m = m" above, which can't be what you mean.) what happens if n = 3 and (a_0, a_1, a_2) = (1,0,0) and (b_0, b_1, b_2) = (0,1,0)? isn't a_0 b_1 + a_1 b_2 + a_2 b_3 = 1*1 + 0*0 + 0*0 = 1 while a_1 b_0 + a_2 b_1 + a_3 b_2 = 0*0 + 0*1 + 1*0 = 0?
 
Mad
nevermind
the statemnt is not true anyway
 
Mad
9:22 AM
let me correct.

$v_{l \pm n} = v_{l }$ where as $v_l$ is a base vector. for $l = 0...n$
i want ot show that the operator give throught $\sum_{l=1}^n v_l \phi_{l-1}$
$\phi_l= <v_l, ->$ is hermetian.
the heremtian of $v_l$ is $\phi_l$ and that of $\phi_{l-1} $ is $v_{l-1}$
This is physics so dont ask about details, none are provided
some hilbert space of quantum mechanical mumbo jumbo
so the hermetian is given through $ \sum_{l=1}^n v_{l-1} \phi_{l}$
Now i need to show that thi is aquivalent. i thought, i show hte sums are the same, which i dont think they are... then, lets try to apply a random base vector $v_j$
but its so confusing with the cyclic indices
 
9:41 AM
0
Q: Show that every closed and bounded set in $\Bbb R$ is compact. Does the same hold in $\Bbb Q$? If not, then give an example.

Thomas FinleyShow that every closed and bounded set in $\Bbb R$ is compact. Does the same hold in $\Bbb Q$? If not, then give an example. This was a strange question, I encountered. I have no problem with the statement that "every closed and bounded set in $\Bbb R$ is compact." This is because, it's immediate...

Hey guys! I need help with this problem.
Are my reasonings correct ?
I dont understand why are they invalid.
I am working on the basic topology of R and as mention, in my OP, that I am not familiar with metric spaces/Topological spaces and stuffs like that.
The definitions of compact sets, I am familiar with are
:"A set $K \subset R$ is compact if every sequence in R has a
convergent subsequence that converges to a point in K." An
alternative definition was suggested that, "A set $K \subset R$ is compact iff it is closed and bounded." Finally, they
said, that these two definitions of compactness are equivalent to this third (alternative) definition as well,
which says, "A set $K \subset R$ is compact iff for every open cover of the set there exists a finite subcover.
I find no errors in my logical structure
The question I asked in the OP is precisely: Is the fact that:"Every subset of $\Bbb Q$ that is closed and bounded in $\Bbb Q$ are compact" is true or not?
I felt that, if we consider the set $A$ to be the concerned particular subset of Q, then we have that all the limit points the set A has, is in Q.
But I think, the fact that I am missing that it is true that all the rational limit points of A are there in Q, but the irrational limit points of A , if at all exists are not in Q.
But then again, we are concerned about compactness in Q,so why should we bother if no irrational limit points is in Q or not?
Is there a definition for compactness for sets in Q? This is because, all I know is the definitions of compactness in R.
Or is the question in my OP itself is ambiguous.
 
10:02 AM
@DanielDonnelly That's very creative! :)
 
When I showed that all rational limit points are in A, so A is definitely closed in Q. Also, A is bounded by a rational. So, can't we say, that A is compact in Q ?
Or in general is this the rule/law/definition that: "A set A, is compact in Q, iff the set is compact in R".
I think this was the fact, that was unknown to me, is it ?
 
10:21 AM
$\Bbb{Q}$ is a metrizable space
A subset of a metric space is compact iff complete and totally bounded.
A closed and bounded set in a metric space need not be compact.
 
@SouravGhosh Sorry, but I have no idea what a metric space is ! I only know the definitions of compactness wrt to a subset in R
 
Do you know "metric subspace"?
or subspace topology?
 
The definitions of compactness,open sets , closed sets , limit points, derived sets, interior points, were introduced to us, with respect to subsets in R
The recommended book was Understanding Analysis by Stephen Abbott
There was a chapter called "Basic Topology in R"
 
How do you define compact set in Q?
 
@Sourav Ghosh The definitions of compact sets, I am familiar with are
:"A set $K \subset R$ is compact if every sequence in R has a
convergent subsequence that converges to a point in K." An
alternative definition was suggested that, "A set $K \subset R$ is compact iff it is closed and bounded." Finally, they
said, that these two definitions of compactness are equivalent to this third (alternative) definition as well,
which says, "A set $K \subset R$ is compact iff for every open cover of the set there exists a finite subcover.
This is my knowledge about compact sets.
 
10:29 AM
Is $[0, 3]\cap \Bbb {Q}$ is compact in $\Bbb Q$?
 
@ThomasFinley the theory is more general than for subsets of R so people often assume your spaces are as general as possible. One generalization is metric spaces, and after that, even more general, topological spaces
 
Can you construct a sequence of rational numbers in $[0, 3]$ that converges to a irrational?
 
So it's good to specify the space you are in
 
@Jakobian the irony is I dont what a space is!
@SouravGhosh my logical sense says yes, if I modify the definitions of compactness( I am familiar with) above
 
Choose the sequence $x_n=(1+1/n) ^n$ for all $n\in \Bbb N$.
 
10:35 AM
@SouravGhosh The sequence converges to e.
That's an irrational number
And that's not in Q
And the sequence lies in the set you mentioned i.e $[0,3]\cap Q$
So, e is a limit point of $[0,3]\cap Q$
But e is not rational, so, e is not in $[0,3]\cap Q$
 
Hence (x_n) doesn't have any convergent subsequence in $\Bbb{Q}$.
 
@SouravGhosh yes!
But then?
@SouravGhosh Does it matter? I am saying this as because, we are dealing with closedness in Q, so as far as the limit points are concerned, if the limit points are rational then they must be in Q
But the fact that "e" is not in $[0,3]\cap Q$ doesn't affect closedness of $[0,3]\cap Q$ as it seems
 
A closed bounded set in $\Bbb{Q}$ need not be compact.
 
oh, did you by any chance mean the definition of the compactness I said, earlier, that:" A set $K \subset R$ is compact if every sequence in K has a
convergent subsequence that converges to a point in K" can be modifued to characterize compact sets in Q as follows that "A set $K \subset Q$ is compact if every sequence in K has a
convergent subsequence that converges to a point in K" and from here, as we saw, that the subsequences of x_n (in the eg you gave) is not in $[0,3]\cap Q$ so, it is not compact in Q.
 
$A\subset \Bbb Q$ is compact iff for every sequence $(x_n) $ in $A$ there is a subsequence $(x_{n_k}) $ that converges to a point in $A$.
 
10:43 AM
Did I get it correctly ?
This implies my understanding was right? I did the same modification, in my previous comment.
 
@SouravGhosh ok, so this is what it boils down to the criteria for compactness in Q, right ?
 
Yes.
 
@SouravGhosh thanks a ton! I remain grateful to you!
 
@ThomasFinley try to produce a sequence in Q that has no convergent subsequence in Q.
i.e. you could try get a sequence in Q that converges to an irrational no. but they are in Q so it does the same job as in my last message.
 
for example, you could take $x_n= 2^{-n} [ \sqrt 2 2^n]$, where [,] is greatest integer function and consider finding $\lim x_n$
note that $x_n\in Q$ for all n but $\lim x_n= \sqrt 2$.
if you don't like that, you can take $x_1=1+1, x_2= 1+1+1/2, x_n=1+1+1/2!+...+1/n!$. All are rational but $\lim x_n= e$ in R.
 
After trying for 2hrs I got the clarification, phew!
 
11:47 AM
@Koro Thanks!
 
Regarding Nico's answer to this post about a proof of the magic diagram being cartesian using the Yoneda embedding:

https://math.stackexchange.com/questions/778186/the-magic-diagram-is-cartesian/4648573#4648573

Why do we need that the functors mentioned preserve limits? Isn't it enough that they reflect them?
oh I see now
I think we want Yoneda to map fiber products to fiber products
in order for Yoneda to map magic squares to magic squares
that's where the continuity is needed
and then we want reflection to get that the new magic square is indeed cartesian
so never mind
 
12:19 PM
Can anyone give me an example of a space X such that every infinite set in X has a limit point in X but X is not sequentially compact?
43
A: Totally bounded, complete $\implies$ compact

Brian M. ScottYou need to show that if $X$ is totally bounded, every sequence in $X$ has a Cauchy subsequence. Let $\sigma=\langle x_n:n\in\mathbb{N}\rangle$ be a sequence in $X$. For each $n\in\mathbb{N}$ let $D_n$ be a finite subset of $X$ such that the open balls of radius $2^{-n}$ centred at the points of ...

how to show that {x_{n_k}: k} is Cauchy?
 
12:35 PM
nvm, I got it now.
 
@Koro I guess one could (should?) modify the construction in the answer slightly to achieve that $d(y_k,y_m)\le\frac1{2^k}$ (for $m\ge k$).
 
I don't think that's required.
$A_n$ is being 'obtained' from $A_{n-1}$ in some way.
 
What I have in mind is using in the definition of $A_{k+1}$ the fact that $\overline{B(y_k,2^{-k})}$ is totally bounded (rather than working with the whole space).
 
Notations are confusing there though.
 
Oh, I missed that $A_{k+1} \subseteq A_k$.
You're right, this should help.
 
12:42 PM
I also didn't see that earlier (before posting my question here) :(.
23 mins ago, by Koro
Can anyone give me an example of a space X such that every infinite set in X has a limit point in X but X is not sequentially compact?
 
Maybe an example of a compact space that is not sequentially compact would work?
Some such examples are listed here: compactness / sequentially compact. (Although the answers/comments are mostly links - but pi-base definitely helps here.)
 
I'll check these out. Thanks a lot :-).
Intuitively they look same so I wanted to see some example distinguishing them.
One has to go beyond metric spaces to realize such a space.
 
@MartinSleziak Yes. A compact space is always limit point compact.
@Koro In a metrizable space, all type of compactness are equivalent.
Topological compact, limit point compact, sequentially compact, countably compact.
Identify the group from the Cayley graph.
 
1:09 PM
I dont understand whether this question is tooooo trivial or am I missing something?
I am talking about the part where it asks to show an example.
If A' is referred to as a complement of A, then it 's trivial, but I bet it's not it. It might be the closure of A ?
Although, if A' represents the closure of a set A, then the question is alright...
What d ya all think?
Oh, wait, if $A'$ refers to the closure of a set A, then as we know the closure of a set is closed, so, $(((A')')...)'=A'$
So I think the criteria that $(A')'\neq \phi,(((A')')')=\phi,$ is never true as $(A')'=(((A')')')$
I want to know what do you guys think about the scenario ?
 
@XanderHenderson I'm still here. I support the strike, but I haven't signed on. So there is still a sheriff around.
 
It's a question from a random handout.
 
1:24 PM
@copper.hat Did you see Farhad's image and my comment?
 
@ThomasFinley it's the set of limit points of A
that is $p\in A'$ iff for any neighbourhood $U$ of $p$, $U\setminus\{p\}\cap A \neq\emptyset$
I can think of a set $A_n\subseteq [0, 1]$ such that $A_n^{(n+1)} = \emptyset$ while $A_n^{(n)} \neq\emptyset$, where $(n)$ denotes taking set of limit points (i.e. derived set) $n$ times
If you care to define what $A^{(\alpha)}$ means for an ordinal $\alpha$, then you can show that for big enough $\alpha$, $A^{(\alpha)}$ is constant, and if $A$ is closed then the set it equals to is the largest set contained in $A$ which is perfect
A set $A$ with $A' = A$ is called perfect
Consider $A_0 = \{0\}$ and $A_1 = \{0\}\cup\{1/n : n\in\mathbb{N}\}$. You can modify this construction to define $A_n$ for any $n$ with desired properties, in fact so that $A_{n+1}' = A_n$
just put a copy of $A_1$ for each $p\in A_{n+1}\setminus A_n$
This will be your $A_{n+2}$
 
Mad
1:52 PM
$
T = \sum_{i=1}^{L} \underbrace{(0,\ldots,0,1,0,\ldots,0)}_{\text{$n$-th position}} \cdot \underbrace{(0,\ldots,1,\ldots,0)}_{\text{$n+1$-th position}}
$

The Matrix is given then as $\begin{bmatrix} 0&1&...\\
0&0&1..\\
.. &.. &.. \\ 1 & 0 & 0
\end{bmatrix}$

if we rearange the matrix rows, we get as a charestic polynomial just $+-(1-\lambda)^L = 0 $ where as L+1 = 1 here
Correct?
so we have basically $(1-\lambda)^L=0$ which only is true if $\lambda = 1$
So the only eigenvalue is 1?
then the eigenvectors is anyvector fullfilling $c_{j+1}^* = c_j$ where as $c_j$ are the vector components in the basis $v_j$
Can someone check my analysis
Thanks
sorry the first vector should be written as a coloumn
 
2:08 PM
X- metric space. $X$ is sequentially compact. $\iff $ fip family of closed sets in it have non empty intersection. $\iff$ X is compact.
$\iff$ every infinite set in X has a limit point. $\iff$ X is complete + totally bounded.
this route doesn't require Lebesgue covering lemma.
Proving $X$ to be compact from X being sequentially compact will require LCL.
but it's probably not necessary.
 
2:35 PM
@Jakobian so what about my reasoning stating that such a set $A$ required in my example does not exist ? Is it valid?
 
Mad
nevernubd u didnt calculate it correcly
 
@Koro ring theory
 
I am talking about this, @Jakobian
1 hour ago, by Thomas Finley
Oh, wait, if $A'$ refers to the closure of a set A, then as we know the closure of a set is closed, so, $(((A')')...)'=A'$
 
@ThomasFinley Jakobian suggested you that such an example exists.
@ThomasFinley A' does NOT refer to the closure of the set. The image you posted also asks you to define derived set. See here:
1 hour ago, by Jakobian
@ThomasFinley it's the set of limit points of A
 
2:52 PM
@Koro Ohh...gosh, now that makes it clear. I missed that lone, and that's why it felt so weird.
 
$cl(A) =A\cup A'$
 
Any simpler examples other than the one that @Jakobian provided ?
It would be very helpful. Such constructions seems heuristical.
 
3:08 PM
Take $A_1$ from Jakobian's comment to start with.
then see if it works. If not, then see if $A_1+A_1$ works.
 
3:30 PM
@robjohn Does that make me Barney Fife?
 
@ThomasFinley Well. Choose $A=\{\frac{1}{m}+\frac{1}{n}:m, n\in\Bbb{N}\}$
Calim : 1) $A'=\{\frac{1}{n}:n\in\Bbb{N}\}\cup\{0\}$
2) $(A')'=\{0\}$
3) $((A')')'=\emptyset$
 
@CroCo Then I don't understand what you're doing. If you're just doing usual row operations, then for the matrix $\begin{bmatrix} a & b\\c & d\end{bmatrix}$ you should get $\begin{bmatrix} t-a & -b\\-c & t-d \end{bmatrix} \rightsquigarrow \begin{bmatrix} 1 & -b/(t-a) \\ 0 & t-d + c(-b/(t-a))\end{bmatrix}$ assuming $t\ne a$. So the matrix is singular iff $t-d-bc/(t-a) = 0$ iff (t-a)(t-d)-bc = 0$. We're just back to the characteristic polynomial.
 
3:53 PM
@SouravGhosh I already left $A_1+A_1$ for the asker to work out on their own but you disturbed all that.
 
@Koro 🦆
 
Good morning, Ted. Soft question... to eliminate further confusions and time-wasting battles in case I ask anything: whenever you don't explicitly say something is incorrect or wrong, it is most likely a suggestion/preference?
 
no it means he's human and may or not be interested in the particular problem you're working on, and may be attending to another matter. why would he owe you any explanation
 
@Koro hehe! The asker already figured it out. Thanks! I liked that example.
 
Let $N$ is a normal subgroup of Gal(L/K).Then there exists a normal extension of $K$ of degree $|N|$.
 
4:04 PM
@SouravGhosh But how the example is worded in here, makes it more precise and to be honest, it's nicely written than my lengthy deductions.
Thanks !
Cheers!
 
Awesome :)
 
@SoumikMukherjee I assume not (a quick Google search yielded nothing at least, which you've probably done too)
 
4:18 PM
Any counter example?
 
I was trying to prove that $(-2,1]$ is not a compact set using the fact that :A set in R is compact iff every cover of the set has a finite subcover. So, I tried generating a cover for $(-2,1]$ corresponding to which, we won't find any subcover. My solution is: We note that, $(-2,1]$ is the union of the sets $(-2,-\frac 1n)$ such that $n\in \Bbb N$ and $(0,2)$ form an open cover for $(-2,1].$ We note that corresponding to this open cover, we don't have a finite subcover.
Is my solution, as it is valid?
 
\cup_{n=1}^{\infty} ( -2, -\frac 1 n) =(-2, -1)
 
@SouravGhosh isn't it (-2,0) ?
I noticed I made a mistake in the last set, i.e (0,2), while it should be (-1/c,2), where $c\in\Bbb N.$
@SouravGhosh or am I missing something big?
 
every open cover has a finite subcover*
 
@Jakobian Yes, so what about my approach ?
 
4:33 PM
If you know that compact sets are closed, you can just see that cl((-2, 1]) = [-2, 1] is so that (-2, 1] is not closed, hence not compact
 
Here's something I don't understand from my lecture notes, which you may freely criticize.
 
@ThomasFinley no it's not.
 
 
@Jakobian I want to prove it specifically using open covers
@Koro why?
 
The point that makes (-2, 1] not compact is -2
so try to adjust your open cover to that point
 
4:35 PM
@sunny Why can we assume $E$ is compact? Moreover, I'd be grateful if someone could clarify $(2)$ and $(3)$? I guess $(1)$ follows from $\left|\int_a^b f(x)\mathrm{d}x\right|\leq\int_a^b |f(x)|\mathrm{d}x$. $(2)$ must simply follow from $|f(x,s)-f(x,s_0)| \leq \mathrm{sup}_{x\in K}|f(x,s)-f(x,s_0)|$, for $s,s_0$ fixed.
$(3)$ confuses me since by uniform continuity we only have $|f(x,s)-f(x,s_0)|< \epsilon$. Why does $\mathrm{sup}_{x\in K} |f(x,s)-f(x,s_0)| <\epsilon$, which I assume is what the last inequality states?
 
because you have not covered (-2,1]
 
yeah, you not covered (-2, 1], but if you would, your type of cover would have a finite subcover, you need to work with point -2 somehow
 
@Koro why? I admit, that the last set "(0,2)" is wrong, it should be $(-1/c,2), $where $c\in\Bbb N.$ but apart from it what goes wrong?
 
take your time. think why.
 
To be precise my open cover collection is: $(-2,-\frac 1n)$ such that $n\in \Bbb N$ and $(-1/c,2), $where $c\in\Bbb N.$
 
4:38 PM
where is 0?
 
In the set, $(-1/c,2), $
 
this open cover has a finite subcover
 
As -1/c\leq 0\leq 2
@Jakobian and what is it?
 
@ThomasFinley I mean here.
 
(-2, -1/2) and (-1, 2) for example
 
4:40 PM
Ohh, no I admitted previously in my comments about the mistakes, 3 times. That why I posted my collection again
In the comment :"To be precise ...."
 
If you're going to take (-2, a) in your cover, then that automatically doesn't work
because if you take b with -2 < b < a, then [b, 1] is compact so on that you can take a finite subcover
 
@Jakobian Ohh...missed it.
If the collection is, $(-2+1/n,2)$ s.t $n\in\Bbb N$ , then I think it might work, yk
 
you have to figure it out yourself
 
@ThomasFinley Do you need hint?
 
@Jakobian Is there any algorithm, to know, whether a collection of open covers has a finite sub cover or not? Or is it just heuristics ?
 
4:49 PM
there's none
 
@SouravGhosh I hate to admit it, but yes, I think it might be helpful...
@Jakobian only heuristics ?
 
@ThomasFinley draw this on a line and finish it.
 
@ThomasFinley wdym by heuristics?
 
@ThomasFinley Yes. If the set (in \Bbb R) is closed and bounded then every open cover has a finite subcover 🦆.
 
(try some values of n)
 
4:51 PM
@SouravGhosh that's not an algorithm
 
@Jakobian proceeding to a solution by trial and error or by rules that are only loosely defined.
 
loosely defined?
 
when trying to prove something is compact, usually people develop theory, and otherwise you have some theorems to help you, when you can take covers by "nice" sets
 
We need to assure existence, not precise subcover.
 
for example what Sourav Ghosh mentioned, for the real line compactness has simple characterization
 
4:53 PM
@Koro it's a copy pasted meaning from google(!)
 
sometimes it gets explicit, other times you can use a theorem
 
@Jakobian Is the golomb space path connected?
 
how can it be path-connected if its countable
 
$(\Bbb N, \tau_{G}) $
@Jakobian Got it.
@Jakobian So totally path disconnected?
 
path connected + Hausdorff implies arc connected
but there can be no arcs in Golomb space
@SouravGhosh yes
 
5:00 PM
@Jakobian Particular point space is path connected
 
but it's not even $T_1$
 
What isnt?
 
@Jakobian I am responding to your previous comment.
 
I know
I said that knowing that Golomb space has good enough separation properties
 
@Jakobian It is $ T_2 $ space.
@Jakobian So a path connected $T_2$ space is uncountable?
 
5:06 PM
If it has more than one point, yes
 
@Jakobian Well. I will try to prove.
 
I think you need Hahn-Mazurkiewicz theorem for that, better not
 
Let $(X,d)$ on compact metric space. Let $(x_n)_n,(y_n)_n$ be sequences in $X$.

Show that or find a counterexample to: there exists a subsequence $(x_{n_k})_k$ and $(y_{n_k})_k$ such that $d(x_{n_k},y_{n_k})\rightarrow 0$
this is false right?
 
Suppose I have a two-variable function $f:\mathbb{R^2}\to\mathbb{R}$ that is uniformly continuous, i.e. given an $\epsilon>0$, I can find a $\delta(\epsilon)$ such that $|f(x,y)-f(x,z)|<\epsilon$ whenever $|x-x|+|y-z|=|y-z|<\delta(\epsilon)$ (I used the $L_1$ and $L_2$ norms here). Does it hold then that $\mathrm{sup}_{x\in\mathbb{R}}|f(x,y)-f(x,z)|<\epsilon$ whenever $|y-z|<\delta(\epsilon)$, assuming $\mathrm{sup}_{x\in\mathbb{R}}|f(x,y)-f(x,z)|$ exists and is finite?
 
I mean it holds if $x_{n_k}$ and $y_{n_k}$ converge to same limit
Otherwise its false right?
 
5:11 PM
@ShaVuklia Yes, that's why I was asking.
 
@Jakobian A path connected $T_1$ space with at least two points is uncountable.
Or we need $T_2$ property?
 
@robjohn Cool beans.
 
@SouravGhosh it seems so, I didn't know of that. But for equivalence between arc-connected and path-connected you need $T_2$
2
 
@monoidaltransform Choose $X=[-1, 1]$ and $x_n=(-1) ^n and $y_n=\frac{1}{n}$
 
telophase topology is $T_1$, path-connected, but not arc-connected
@monoidaltransform take a space with at least two points and two constant sequences
 
5:17 PM
@Jakobian Wow!
 
Ah, this follows from Sierpiński theorem
 
@monoidaltransform I don't even know what this means. You certainly should say $\sup\le\epsilon$, right?
 
You cannot partition $[0, 1]$ into infinitely countable amount of closed sets, where a path $f:[0, 1]\to X$ into $T_1$ space $X$ is such that $f^{-1}(x), x\in f([0, 1])$ forms a partition of $[0, 1]$ into closed sets
 
@noballpointpen I believe that I say things are wrong when they are, or at least I ask a leading question that suggests that.
 
5:39 PM
@Jakobian Nice:)
$[0, 1]$ can't be an $F_{\sigma}$ set.
A nice application of the Baire's theorem.
A complete metric space without any isolated point is uncountable.
 
it is an $F_\sigma$ set
$[0, 1]$ is $\sigma$-connected
is the term
 
My bad. Closed set must be $F_{\sigma}$.
 
this is a property of continua in general, I believe, but there are connected sets which aren't $\sigma$-connected
 
@Jakobian What about:" If the collection is, $(-2+1/n,2)$ s.t $n\in\Bbb N$ then it covers (-2,1]" ? As i said once in my prev comment. Did I figure it correctly ?
I feel yes
 
it does cover it, it's an open cover in fact
but what you wanted to show it that it doesn't have a finite subcover
 
5:48 PM
@Jakobian What is the definition of "sigma connected" space?
 
@SouravGhosh that there's no partition into $2\leq k\leq \aleph_0$ closed sets
2
so the theorem is just that $[0, 1]$ is $\sigma$-connected
 
@Jakobian and it literally doesn't have a finite subcover, is it? I can't find any, though
 
@Jakobian Nice.
 
@ThomasFinley if it doesn't then justify it somehow
 
@Jakobian Done! $[0, 1]=\cup_{n\in\Bbb{N}} C_n$ where $C_n$'s are closed disjoint sets. Let $E_n=\bdd(C_n) $. Then $E=\cup_n E_n$ .
Then we apply Baire's theorem on $E$ to conclude that $E$ is uncountable.
 
 
1 hour later…
7:22 PM
What do you feel is the most important thing when giving a math talk?
("Thing" is purposely left undefined)
 
7:38 PM
besides ending on time?
 
bluntly responding: i don't think your question is relevant to my talk whenever an irrelevant question comes up at the end
quickly follow up with: are there any other questions
afterwards try to get in a fight with that person
 
do a 1vs 1 death battle
and then do proof by physical superiority
if there is no other mathematicians to say you're wrong, that makes you right
 
besides ending on time, i think a talk should do something that isn't already done (or about to be done) in a paper. it doesn't have to involve the audience, it can be entirely one-way, but it shouldn't just recap something that someone can go look up. so many bad talks are "here's a talk on my latest paper, in the interest of time i will only give a brief statement of the result and the proof of the paper's lemma 2.1."
 
DO EXAMPLES! My over-riding advice.
 
talks are a safe space for speaking vaguely about where the ideas came from, speaking about things that you might try but don't work, and saying stronger things than you might in a paper (e.g. "there's really only one idea in this, and it's the following")
 
7:44 PM
tirelessly try to discern whether the question asker is trying to trap you or has a hidden agenda
 
so even if a talk is very much "about" or even limited to a recent paper, it's nice if the talk can provide what feels like an "inside look" at how the paper came to be, including things that would never be put in print for space reasons or other reasons.
 
your reputation is at stake, so should his be
 
as shintuku notes, a talk should also touch on at least some aspects of current foreign policy, ideally that of the united states
 
Ted....I'm doind the question that asks to show $I-H$ is inveritble, using the geometric series $\sum_{k=0}^\infty H^k$
 
Yes?
 
7:48 PM
as leslie notes, make sure your talk is a safe space for you and no one else. the only acceptable question askers are those who ask in fear
and yeah recognize the hegemonic status of the us but add in a little critical tidbit to please everyone
 
I considered $\|\sum_{k=0}^\infty H^k\| \leq \dots \leq\|H\| \sum_{i=0}^\infty H^i = \frac{\|H\|}{1 - \|H\|}$
 
Consider the two vectors $k_1 \hat u=k_2 \vec{N}$. If $|| \vec{N}||=1$. can I say that $k_1=k_2$?
 
so I have this now, but how would I use this to show inveritbility?
 
$\hat u$ is a unit vector .
 
unknown: if u and N are unit vectors, you can deduce that |k_1| = |k_2|, which in the real case means k_1 = k_2 or k_1 = -k_2, and in the complex case you have more possibilities
 
7:52 PM
I have to get a map $B$ such that $B(I-H) = 1$. Also in the dots I used the idea of the hint from exercise 5.1.6: \|AB\| \leq \|A\| \|B\|$
 
@leslietownes Yes. Now I understood. why did the author say $k_1=k_2$ in the proof of fundamental theorem of curve theory. Thanks
 
i hate to say it, but maybe there is some geometric reason why u and N point in the same direction, which would rule out k_1 and k_2 having opposite sign
 
@leslietownes No actually, $k_1$ and $k_2$ are curvatures. so. it have to be same.
 

« first day (4691 days earlier)      last day (332 days later) »